LSAT and Law School Admissions Forum

Get expert LSAT preparation and law school admissions advice from PowerScore Test Preparation.

 smile913
  • Posts: 6
  • Joined: Aug 31, 2016
|
#28206
Hi,

I'm having a hard time understanding why (C) is the correct answer. The passage simply states that "Clues are presented in the story" and "the author's strategy of including a dull companion gives readers a chance to solve the mystery". From the given statements, how can we conclude that some mystery stories give readers enough clues? Isn't (C) a generalization of the original statements?

Your reply would be greatly appreciated.
Thank you.
User avatar
 Dave Killoran
PowerScore Staff
  • PowerScore Staff
  • Posts: 5852
  • Joined: Mar 25, 2011
|
#28207
Hi Smile,

Thanks for the question! to broadly summarize, it's actually this phrase that most leads us to (C): "the author’s strategy of including the dull companion gives readers a chance to solve the mystery" (italics added). From the earlier phrase, we know that clues are provided, but by itself we can't say those clues are sufficient to allow the mystery to be solved (although we know those clues lead the dull companion astray). But, in the conclusion, the author indicates that readers do have a chance to solve the mystery, and this means at least one story has enough clues to allow the reader to solve the mystery. As you will see in more detail later, "at least one" is equivalent to "some," and thus answer choice (C) is supported.

Please let me know if that helps. Thanks!
 smile913
  • Posts: 6
  • Joined: Aug 31, 2016
|
#28208
Dear Mr. Killoran,

Thank you so much for the reply! I finally understood why (C) is the correct answer.
Your explanation was awesome. Thank you.

Get the most out of your LSAT Prep Plus subscription.

Analyze and track your performance with our Testing and Analytics Package.